Predictions that printed books will soon be replaced by books in electronic formats such as CD-ROM are exaggerated. W...

jonah-Mead-Vancort on November 4, 2019

answer d

answer d seems to be the logical fallacy that is the missing link of the argument.

Replies
Create a free account to read and take part in forum discussions.

Already have an account? log in

Annie on November 8, 2019

@jonah-Mead-Vancort,

This question is asking you to find the answer that "most strengthens" the argument. Therefore, you're looking for an answer choice that adds a premise which will help support the conclusion. Here's a break down of the argument:

Premise: While research libraries may find electronic books more convenient, bookstores and public libraries will stock books in the format desired by the general public.
Premise: The format desired by the general public is something other than electronic books.
Conclusion: Predictions that printed books will be replaced by electronic books are exaggerated.

Answer Choices:
(A) is incorrect. This answer choice supports premise 1 but does not support the conclusion that these predictions are exaggerated.

(B) is correct. This answer choice adds another premise to the mix. It states that publishers will keep printing books in the format desired by bookstores and public libraries. We know from the other premises that bookstores and public libraries will stock books in the format desired by the public, which is not electronic. Therefore, this answer choice supports the conclusion that electronic books are not going to wipe out printed books, as it tells us that publishers will keep printing in other forms.

(C) is incorrect. This answer choice does not necessarily have an effect on printed books. All it tells us is that scholars and scientists are unlikely to frequent public libraries. However, we have no information regarding how a big of a market scholars and scientists are. If the rest of the public continues to attend public libraries, we likely still have a strong market for printed books.

(D) is incorrect. This answer choice tells us nothing about the competition between electronic books and printed books, which is the focus of the argument. Audio-books and videos are a different subset, and therefore this answer choice is irrelevant.

(E) is incorrect. We are looking for an answer choice that strengthens the argument, this one weakens it and is therefore incorrect.

Annie on November 8, 2019

@jonah-Mead-Vancort,

This question is asking you to find the answer that "most strengthens" the argument. Therefore, you're looking for an answer choice that adds a premise which will help support the conclusion. Here's a break down of the argument:

Premise: While research libraries may find electronic books more convenient, bookstores and public libraries will stock books in the format desired by the general public.
Premise: The format desired by the general public is something other than electronic books.
Conclusion: Predictions that printed books will be replaced by electronic books are exaggerated.

Answer Choices:
(A) is incorrect. This answer choice supports premise 1 but does not support the conclusion that these predictions are exaggerated.

(B) is correct. This answer choice adds another premise to the mix. It states that publishers will keep printing books in the format desired by bookstores and public libraries. We know from the other premises that bookstores and public libraries will stock books in the format desired by the public, which is not electronic. Therefore, this answer choice supports the conclusion that electronic books are not going to wipe out printed books, as it tells us that publishers will keep printing in other forms.

(C) is incorrect. This answer choice does not necessarily have an effect on printed books. All it tells us is that scholars and scientists are unlikely to frequent public libraries. However, we have no information regarding how a big of a market scholars and scientists are. If the rest of the public continues to attend public libraries, we likely still have a strong market for printed books.

(D) is incorrect. This answer choice tells us nothing about the competition between electronic books and printed books, which is the focus of the argument. Audio-books and videos are a different subset, and therefore this answer choice is irrelevant.

(E) is incorrect. We are looking for an answer choice that strengthens the argument, this one weakens it and is therefore incorrect.